0
$\begingroup$

Asking for help in approaching a question from a Statistics textbook:

Let $X_1, X_2, ..., X_n$ independent and identically distributed with density function $f_ {\theta}(x)$ and $T_n(X_1,X_2,...X_n)$ a sufficient statistic for $\theta$ that is a continuous random variable. Let $g_{\theta}(t)=F_{T_n,\theta}(t)=\mathbb{P}_{\theta}(T_n\leq t)$ be the cumulative distribution function of $T_n$, and $Y_n=g(T_n)$. Show that $Y_n\sim U(0,1)$ and thus $Y_n$ is a pivot.

My initial thoughts were to somehow find the distribution of $T_n$ using a change of variables, but $T_n$ is not the same dimension as the probability space of the sample $X_1,..X_n$. Or to somehow use the sufficiency to write $f_{\theta}(x)=h(x)g_{\theta}(T_n(x))$, not sure though how to proceed from here.

$\endgroup$
3
  • 2
    $\begingroup$ The beginning is useless. As soon as $T_n$ is a continuous random variable, and $g$ is its cumulative distribution function, the composition $g(T_n)$ is uniform over $(0,1)$. $\endgroup$ Commented Oct 24 at 7:05
  • $\begingroup$ see e,g, chatgpt.com/s/t_68fb26d41430819191b10be64141e56c for the proof.... $\endgroup$ Commented Oct 24 at 7:12
  • $\begingroup$ You seem to need $T_n$ to be one dimensional for $\mathbb{P}_{\theta}(T_n\leq t)$ to make sense. $\endgroup$ Commented Oct 24 at 9:01

1 Answer 1

0
$\begingroup$

Thanks for the comments. $Y_n\sim U(0,1)$ is a consequence of the probability integral transform, and $T$ being a sufficienct statistic is irrelevant - only that it's a continuous variable.

$\endgroup$

You must log in to answer this question.

Start asking to get answers

Find the answer to your question by asking.

Ask question

Explore related questions

See similar questions with these tags.